Saltar al contenido principal
LibreTexts Español

4.6: Funciones convexas y derivados

  • Page ID
    107816
  • \( \newcommand{\vecs}[1]{\overset { \scriptstyle \rightharpoonup} {\mathbf{#1}} } \) \( \newcommand{\vecd}[1]{\overset{-\!-\!\rightharpoonup}{\vphantom{a}\smash {#1}}} \)\(\newcommand{\id}{\mathrm{id}}\) \( \newcommand{\Span}{\mathrm{span}}\) \( \newcommand{\kernel}{\mathrm{null}\,}\) \( \newcommand{\range}{\mathrm{range}\,}\) \( \newcommand{\RealPart}{\mathrm{Re}}\) \( \newcommand{\ImaginaryPart}{\mathrm{Im}}\) \( \newcommand{\Argument}{\mathrm{Arg}}\) \( \newcommand{\norm}[1]{\| #1 \|}\) \( \newcommand{\inner}[2]{\langle #1, #2 \rangle}\) \( \newcommand{\Span}{\mathrm{span}}\) \(\newcommand{\id}{\mathrm{id}}\) \( \newcommand{\Span}{\mathrm{span}}\) \( \newcommand{\kernel}{\mathrm{null}\,}\) \( \newcommand{\range}{\mathrm{range}\,}\) \( \newcommand{\RealPart}{\mathrm{Re}}\) \( \newcommand{\ImaginaryPart}{\mathrm{Im}}\) \( \newcommand{\Argument}{\mathrm{Arg}}\) \( \newcommand{\norm}[1]{\| #1 \|}\) \( \newcommand{\inner}[2]{\langle #1, #2 \rangle}\) \( \newcommand{\Span}{\mathrm{span}}\)\(\newcommand{\AA}{\unicode[.8,0]{x212B}}\)

    Discutimos en esta sección una clase de funciones que juega un papel importante en los problemas de optimización.

    clipboard_e7de8eeac8e3a14983f5df875de0911e3.png

    Figura\(4.6\): Una función convexa.

    Definición\(\PageIndex{1}\)

    Dejar\(I\) ser un intervalo de\(\mathbb{R}\) y dejar\(f: I \rightarrow \mathbb{R}\). Decimos que\(f\) es convexo sobre\(I\) si\[f(\lambda u+(1-\lambda) v) \leq \lambda f(u)+(1-\lambda) f(v)\] para todos\(u, v \in I\) y para todos\(\lambda \in (0, 1)\).

    Ejemplo\(\PageIndex{1}\)

    Las siguientes funciones son convexas.

    1. \(f: \mathbb{R} \rightarrow \mathbb{R}\),\(f(x)=x\).
    2. \(f: \mathbb{R} \rightarrow \mathbb{R}\),\(f(x)=x^{2}\).
    3. \(f: \mathbb{R} \rightarrow \mathbb{R}\),\(f(x)=|x|\).

    Solución

    1. Esto es sencillo.
    2. Aquí no primero eso\(2 x y \leq x^{2}+y^{2}\) para todos los números reales\(x\),\(y\). Entonces, si\(0<\lambda<1\) y\(x, y \in \mathbb{R}\), obtenemos\ [\ begin {alineado}
      f (\ lambda x+ (1-\ lambda) y) & =(\ lambda x+ (1-\ lambda) y) ^ {2}\\
      &=\ lambda^ {2} x^ {2} +2\ lambda (1-\ lambda) x y+ (1-\ lambda) ^ {2} y^ {2}\\
      & leq\ lambda^ {2} x^ {2} +\ lambda (1-\ lambda)\ izquierda (x ^ {2} +y^ {2}\ derecha) + (1-\ lambda) ^ {2} y^ {2}\\
      &=\ lambda\ izquierda (\ lambda x^ {2} + (1-\ lambda) x^ {2}\ derecha) + (1-\ lambda)\ izquierda (\ lambda y^ {2} + (1-\ lambda) y^ {2}\ derecha)\
      &=\ lambda x^ {2} + (1-\ lambda) y^ {2}\\
      &=\ lambda f (x) + (1-\ lambda) f (y)\ text {.}
      \ end {alineado}\]
    3. Esto se desprende de la desigualdad triangular y otras propiedades básicas de valor absoluto.

    Teorema\(\PageIndex{1}\)

    Dejar\(I\) ser un intervalo de\(\mathbb{R}\). Una función\(f: I \rightarrow \mathbb{R}\) es convexa si y solo si para cada\(\lambda_{i} \geq 0, i=1, \ldots, n\), con\(\sum_{i=1}^{n} \lambda_{i}=1\)\((n \geq 2)\) y para cada\(x_{i} \in I\)\(\i=1, \dots, n\),\[f\left(\sum_{i=1}^{n} \lambda_{i} x_{i}\right) \leq \sum_{i=1}^{n} \lambda_{i} f\left(x_{i}\right) .\]

    Prueba

    Dado que lo contrario se sostiene trivialmente, solo necesitamos demostrar esa implicación por inducción. Las conclusiones se sostienen\(n = 2\) por la definición de convexidad. \(k\)Sea tal que la conclusión se mantenga para cualquiera\(n\) con\(2 \leq n \leq k\). Demostraremos que también se sostiene para\(n = k + 1\). Arreglar\(\lambda_{i} \geq 0, i=1, \ldots, k+1\), con\(\sum_{i=1}^{k+1} \lambda_{i}=1\) y arreglar cada\(x_{i} \in I, i=1, \ldots, k+1\). Entonces\[\sum_{i=1}^{k} \lambda_{i}=1-\lambda_{k+1} .\] Si\(\lambda_{k+1}=1\), entonces\(\lambda_{i}=0\) para todos\(i=1, \ldots, k\), y (4.6.2) sostiene. Supongamos\(0 \leq \lambda_{k+1}<1\). Entonces, para cada uno\(i=1, \ldots, k\),\(\lambda_{i} /\left(1-\lambda_{k+1}\right) \geq 0\) y\[\sum_{i=1}^{k} \frac{\lambda_{i}}{1-\lambda_{k+1}}=1 .\] se deduce que\ [\ begin {alineado}
    f\ left (\ sum_ {i=1} ^ {k+1}\ lambda_ {i} x_ {i}\ right) &=f\ left [\ left (1-\ lambda_ {k+1}\ right)\ frac {\ sum_ {i=1} ^ {k}\ lambda_ {i} x_ {i}} {1-\ lambda_ {k+1}} +\ lambda_ {k+1} x_ {k+1}\ derecha]\\
    & amp;\ leq\ izquierda (1-\ lambda_ {k+1}\ derecha) f\ izquierda (\ frac {\ sum_ {i=1} ^ {k}\ lambda_ {i} x_ {i}} {1-\ lambda_ {k+1}}\ derecha) +\ lambda_ {k+1} f\ izquierda (x_ {k+1}\ derecha)\
    &=\ izquierda (1-\ lambda_ {k+1}\ derecha) f\ izquierda (\ sum_ {i=1} ^ {k}\ frac {\ lambda_ {i}} {1-\ lambda_ {k+1}} x_ {i}\ derecha) +\ lambda_ {k+1} f\ izquierda (x_ {k+1}\ derecha)\\
    &\ leq\ izquierda (1-\ lambda_ {k+1}\ derecha)\ sum_ {i=1} ^ {k}\ frac {\ lambda_ {i}} {1-\ lambda_ {k+1}} f\ izquierda (x_ {i}\ derecha) +\ lambda_ {k+1} f\ izquierda (x_ {k+1}\ derecha)\\
    &= suma_ {i=1} ^ {k+1}\ lambda_ {i} f\ izquierda (x_ {i}\ derecha),
    \ end {alineada}\] donde la primera desigualdad se desprende de la definición de convexidad (o es trivial si\(\lambda_{k+1}=0\)) y la última desigualdad se deriva de la suposición inductiva. La prueba ya está completa. \(\square\)

    Teorema\(\PageIndex{2}\)

    Dejar\(I\) ser un intervalo y dejar\(f: I \rightarrow \mathbb{R}\) ser una función convexa. Entonces\(f\) tiene un mínimo local en\(\bar{x}\) si y solo si\(f\) tiene un mínimo absoluto en\(\bar{x}\).

    Prueba

    Claramente si\(f\) tiene un mínimo global en\(\bar{x}\), entonces también tiene un mínimo local en\(\bar{x}\).

    Por el contrario, supongamos que\(f\) tiene un mínimo local en\(\bar{x}\). Entonces existe\(\delta > 0\) tal que\[f(u) \geq f(\bar{x}) \text { for all } u \in B(\bar{x} ; \delta) \cap I .\] Para cualquiera\(x \in I\), tenemos\(x_{n}=\left(1-\frac{1}{n}\right) \bar{x}+\frac{1}{n} x \rightarrow \bar{x}\). Así,\(x_{n} \in B(\bar{x} ; \delta) \cap I\) cuando\(n\) es suficientemente grande. Así, para tal\(n\),\[f(\bar{x}) \leq f\left(x_{n}\right) \leq\left(1-\frac{1}{n}\right) f(\bar{x})+\frac{1}{n} f(x) .\] Esto implica que para un suficientemente grande\(n\), tenemos\[\frac{1}{n} f(\bar{x}) \leq \frac{1}{n} f(x)\] y, de ahí,\(f(\bar{x}) \leq f(x)\). Ya que\(x\) fue arbitrario, esta muestra\(f\) tiene un mínimo absoluto en\(\bar{x}\). \(square\)

    Teorema\(\PageIndex{3}\)

    Dejar\(I\) ser un intervalo abierto y dejar\(f: I \rightarrow \mathbb{R}\) ser una función convexa. Supongamos que\(f\) es diferenciable en\(\bar{x}\). Entonces\[f^{\prime}(\bar{x})(x-\bar{x}) \leq f(x)-f(\bar{x}) \text { for all } x \in I .\]

    Prueba

    Para cualquiera\(x \in I\) y\(t \in (0, 1)\), tenemos\ [\ begin {alineado}
    \ frac {f (\ bar {x} +t (x-\ bar {x})) -f (\ bar {x})} {t} &=\ frac {f (t x+ (1-t)\ bar {x}) -f (\ bar {x})} {t}\\
    &\ leq\ frac {t f (x) + (1-t) f (\ bar {x}) -f (\ bar {x})} {t}\\
    &=f (x) -f (\ bar {x})\ texto {.}
    \ end {aligned}\] Dado que\(f\) es diferenciable en\(\bar{x}\),\[f^{\prime}(\bar{x})(x-\bar{x})=\lim _{t \rightarrow 0^{+}} \frac{f(\bar{x}+t(x-\bar{x}))-f(\bar{x})}{t} \leq f(x)-f(\bar{x}) ,\] lo que completa la prueba. \(\square\)

    Corolario\(\PageIndex{4}\)

    Dejar\(I\) ser intervalo abierto y dejar\(f: I \rightarrow \mathbb{R}\) ser una función convexa. Supongamos que\(f\) es diferenciable en\(\bar{x}\). Entonces\(f\) tiene un mínimo absoluto en\(\bar{x}\) si y solo si\(f^{\prime}(\bar{x})=0\).

    Prueba

    Supongamos que\(f\) tiene un mínimo absoluto en\(\bar{x}\). Por Teorema 4.2.1,\(f^{\prime}(\bar{x})=0\). Demostremos lo contrario. Supongamos\(f^{\prime}(\bar{x})=0\). Se deduce del Teorema 4.6.3 que\[0=f^{\prime}(\bar{x})(x-\bar{x}) \leq f(x)-f(\bar{x}) \text { for all } x \in I .\] Esto implica\[f(x) \geq f(\bar{x}) \text { for all } x \in I .\] Así,\(f\) tiene un mínimo absoluto en\(\bar{x}\). \(\square\)

    Lema\(\PageIndex{5}\)

    Dejar\(I\) ser un intervalo abierto y supongamos que\(f: I \rightarrow \mathbb{R}\) es una función convexa. Arreglar\(a, b, x \in I\) con\(a < x < b\). Entonces\[\frac{f(x)-f(a)}{x-a} \leq \frac{f(b)-f(a)}{b-a} \leq \frac{f(b)-f(x)}{b-x} .\]

    Prueba

    Dejar\[t=\frac{x-a}{b-a} .\] Entonces\(t \in (0, 1) \) y\[f(x)=f(a+(x-a))=f\left(a+\frac{x-a}{b-a}(b-a)\right)=f(a+t(b-a))=f(t b+(1-t) a) .\] Por convexidad de\(f\), obtenemos\[f(x) \leq t f(b)+(1-t) f(a) .\] Así,\[f(x)-f(a) \leq t f(b)+(1-t) f(a)-f(a)=t[f(b)-f(a)]=\frac{x-a}{b-a}(f(b)-f(a)) .\] Equivalentemente,\[\frac{f(x)-f(a)}{x-a} \leq \frac{f(b)-f(a)}{b-a} .\] Similarmente, De\[f(x)-f(b) \leq t f(b)+(1-t) f(a)-f(b)=(1-t)[f(a)-f(b)]=\frac{x-b}{b-a}[f(b)-f(a)] .\] ello se deduce que\[\frac{f(b)-f(a)}{b-a} \leq \frac{f(b)-f(x)}{b-x} .\] La prueba ya está completa. \(\square\)

    Teorema\(\PageIndex{6}\)

    Dejar\(I\) ser un intervalo abierto y dejar\(f: I \rightarrow \mathbb{R}\) ser una función diferenciable. Entonces\(f\) es convexo si y sólo si\(f^{\prime}\) va aumentando en\(I\).

    Prueba

    Supongamos\(f\) es convexo. Arreglar\(a < b\) con\(a, b \in I\). Por Lema 4.6.5, para cualquier\(x \in (a, b)\), tenemos\[\frac{f(x)-f(a)}{x-a} \leq \frac{f(b)-f(a)}{b-a} .\] Esto implica, tomar límites, que\[f^{\prime}(a) \leq \frac{f(b)-f(a)}{b-a} .\] De igual manera,\[\frac{f(b)-f(a)}{b-a} \leq f^{\prime}(b) .\] Por lo tanto\(f^{\prime}(a) \leq f^{\prime}(b)\),, y\(f^{\prime}\) es una función creciente.

    Demostremos lo contrario. Supongamos que\(f^{\prime}\) es increasig. Fijar\(x_{1}<x_{2}\) y\(t \in (0, 1)\). Entonces\[x_{1}<x_{t}<x_{2} ,\] donde\(x_{t}=t x_{1}+(1-t) x_{2}\). Por el Teorema del Valor Medio (Teorema 4.2.3), existe\(c_{1}\) y\(c_{2}\) tal que\[x_{1}<c_{1}<x_{t}<c_{2}<x_{2}\] con\ [\ begin {array} {l}
    f\ left (x_ {t}\ right) -f\ left (x_ {1}\ right) =f^ {\ prime}\ left (c_ {1}\ right)\ left (x_ {t} -x_ {1}\ right) =f^ {prime}\ izquierda (c_ {1}\ derecha) (1-t)\ izquierda (x_ {2} -x_ {1}\ derecha)\ texto {;}\
    f\ izquierda (x_ {t}\ derecha) -f\ izquierda (x_ {2}\ derecha) =f^ {\ prime}\ izquierda (c_ {2}\ derecha)\ izquierda (x_ {t} -x_ {2}\ derecha) =f^ {\ prime}\ izquierda (c_ {2}\ derecha) t\ izquierda (x_ {1} -x_ {2}\ derecha)\ text {.}
    \ end {array}\] l Esto implica\ [\ begin {array} {l}
    t f\ left (x_ {t}\ right) -t f\ left (x_ {1}\ right) =f^ {\ prime}\ left (c_ {1}\ right) t (1-t)\ left (x_ {2} -x_ {1}\ right)\ text {;}\
    (1-t) f\ izquierda (x_ {t}\ derecha) - (1-t) f\ izquierda (x_ {2}\ derecha) =f^ {\ prime}\ izquierda (c_ {2}\ derecha) t (1-t)\ izquierda (x_ {1} -x_ {2}\ derecha)\ texto {.}
    \ end {array}\] Ya que\(f^{\prime}\left(c_{1}\right) \leq f^{\prime}\left(c_{2}\right)\), tenemos\[t f\left(x_{t}\right)-t f\left(x_{1}\right)=f^{\prime}\left(c_{1}\right) t(1-t)\left(x_{2}-x_{1}\right) \leq f^{\prime}\left(c_{2}\right) t(1-t)\left(x_{2}-x_{1}\right)=(1-t) f\left(x_{2}\right)-(1-t) f\left(x_{t}\right) .\] reordenando términos, obtenemos\[f\left(x_{t}\right) \leq t f\left(x_{1}\right)+(1-t) f\left(x_{2}\right) .\] Por lo tanto,\(f\) es convexo. La prueba ya está completa. \(\square\)

    Corolario\(\PageIndex{7}\)

    Dejar\(I\) ser un intervalo abierto y dejar\(f: I \rightarrow \mathbb{R}\) ser una función. Supongamos que\(f\) es dos veces diferenciable en\(I\). Entonces\(f\) es convexo si y solo si\(f^{\prime \prime}(x) \geq 0\) para todos\(x \in I\).

    Prueba

    De la Proposición 4.3.2 se deduce que\(f^{\prime \prime}(x) \geq 0\) para todos\(x \in I\) si y sólo si la función derivada\(f^{\prime}\) va aumentando sobre\(I\). La conclusión luego sigue directamente del Teorema 4.6.6. \(\square\)

    Ejemplo\(\PageIndex{2}\)

    Considera la función\(f: \mathbb{R} \rightarrow \mathbb{R}\) que le da\(f(x)=\sqrt{x^{2}+1}\).

    Solución

    Ahora,\(f^{\prime}(x)=x / \sqrt{x^{2}+1}\) y\(f^{\prime \prime}(x)=1 /\left(x^{2}+1\right)^{3 / 2}\). Ya que\(f^{\prime \prime}(x) \geq 0\) para todos\(x\), se desprende del corolario que\(f\) es convexo.

    Teorema\(\PageIndex{8}\)

    Dejar\(I\) ser un intervalo abierto y dejar\(f: I \rightarrow \mathbb{R}\) ser una función convexa. Entonces es localmente Lipschitz continuo en el sentido de que para cualquiera\(\bar{x} \in I\), existe\(\ell \geq 0\) y\(\delta > 0\) tal que\[|f(u)-f(v)| \leq \ell|u-v| \text { for all } u, v \in B(\bar{x} ; \delta) .\] en particular,\(f\) es continuo.

    Prueba

    Arreglar cualquier\(\bar{x} \in I\). Elige cuatro números\(a, b, c, d\) satisfactorios\[a<b<\bar{x}<c<d \text { with } a, d \in I .\] Elige\(\delta > 0\) tal que\(B(\bar{x} ; \delta) \subset(b, c)\). Déjalo\(u, v \in B(\bar{x} ; \delta)\) con\(v < u\). Entonces por Lemma 4.6.5, vemos que\[\frac{f(b)-f(a)}{b-a} \leq \frac{f(u)-f(a)}{u-a} \leq \frac{f(u)-f(v)}{u-v} \leq \frac{f(d)-f(v)}{d-v} \leq \frac{f(d)-f(c)}{d-c} .\] Usando un enfoque similar para el caso\(u < v\), obtenemos\[\frac{f(b)-f(a)}{b-a} \leq \frac{f(u)-f(v)}{u-v} \leq \frac{f(d)-f(c)}{d-c} \text { for all } u, v \in B(\bar{x} ; \delta) .\] Elegir\(\ell \geq 0\) suficientemente grande para que\[-\ell \leq \frac{f(b)-f(a)}{b-a} \leq \frac{f(u)-f(v)}{u-v} \leq \frac{f(d)-f(c)}{d-c} \leq \ell \text { for all } u, v \in B(\bar{x} ; \delta) .\] Entonces (4.6.29 sostiene. La prueba ya está completa. \(\square\)

    Ejercicio\(\PageIndex{1}\)

    1. Dejar\(I\) ser un intervalo y dejar\(f, g: I \rightarrow \mathbb{R}\) ser funciones convexas. Demostrar que\(cf\)\(f + g\),, y\(\max \{f, g\}\) son funciones convexas en\(I\), donde\(c \geq 0\) es una constante.
    2. Encuentra dos funciones convexas\(f\) y\(g\) en un intervalo\(I\) tal que no\(f \cdot g\) sea convexo.
    Contestar

    Agrega textos aquí. No borre primero este texto.

    Ejercicio\(\PageIndex{2}\)

    Dejar\(f: \mathbb{R} \rightarrow \mathbb{R}\) ser una función convexa. Dado\(a, b \in \mathbb{R}\), demostrar que la función definida por\[g(x)=f(a x+b), \text { for } x \in \mathbb{R}\] es también una función convexa en\(\mathbb{R}\).

    Contestar

    Agrega textos aquí. No borre primero este texto.

    Ejercicio\(\PageIndex{3}\)

    Dejar\(I\) ser un intervalo y dejar\(f: I \rightarrow \mathbb{R}\) ser una función convexa. Supongamos que\(\phi\) es una función convexa, creciente en un intervalo\(J\) que contiene\(f(I)\). Demostrar que\(\phi \circ f\) es convexo en\(I\).

    Contestar

    Agrega textos aquí. No borre primero este texto.

    Ejercicio\(\PageIndex{4}\)

    Demostrar que cada una de las siguientes funciones es convexa en el dominio dado:

    1. \(f(x)=e^{b x}\),\(x \in \mathbb{R}\), donde\(b\) es una constante.
    2. \(f(x)=x^{k}\),\(x \in[0, \infty)\) y\(k \geq 1\) es una constante.
    3. \(f(x)=-\ln (1-x)\),\(x \in(-\infty, 1)\).
    4. \(f(x)=-\ln \left(\frac{e^{x}}{1+e^{x}}\right)\),\(x \in \mathbb{R}\).
    5. \(f(x)=x \sin x\),\(x \in\left(-\frac{\pi}{4}, \frac{\pi}{4}\right)\).
    Contestar

    Agrega textos aquí. No borre primero este texto.

    Ejercicio\(\PageIndex{5}\)

    Demostrar lo siguiente:

    1. Si\(a, b\) son números reales no negativos, entonces\[\frac{a+b}{2} \geq \sqrt{a b} .\]
    2. Si\(a_{1}, a_{2}, \ldots, a_{n}\), donde\(n \geq 2\), son números reales no negativos, entonces\[\frac{a_{1}+a_{2}+\cdots+a_{n}}{n} \geq\left(a_{1} \cdot a_{2} \cdots a_{n}\right)^{1 / n} .\]
    Contestar

    Agrega textos aquí. No borre primero este texto.


    This page titled 4.6: Funciones convexas y derivados is shared under a CC BY-NC-SA license and was authored, remixed, and/or curated by Lafferriere, Lafferriere, and Nguyen (PDXOpen: Open Educational Resources) .